0
$\begingroup$

Let $p(x)$ be a probability density function on the unbounded set $X \subseteq \mathbb{R}^n$, so that $\int_X p(x) dx = 1$.

Let $F: X \rightarrow \mathbb{R}_{\geq 0}$ a measurable but non-integrable function, i.e.

$$ \int_X F(x) p(x) dx = \infty $$

where $X \subseteq \mathbb{R}^n$ is a closed unbounded set.

I'm wondering if the following is true:

$$ \forall \text{ such } F(\cdot) \ \ \exists \text{ a strictly-increasing, concave } f: \mathbb{R}_{\geq 0} \rightarrow \mathbb{R}_{\geq 0} \text{ with } f(0)=0 \text{ such that: } \\ \int_X f(F(x)) p(x) dx < \infty $$

Is it true if, in addition, we require $\lim_{x \rightarrow \infty} f(x) = +\infty$?

  • 2
    Any bounded increasing concave function $f$ such that $f(0)=0$ solves the problem, for example $f:t\mapsto t/(1+t)$.2012-04-02
  • 0
    Alternatively, it might be possible to pick any strictly-increasing, concave function $g(x)$ that is integrable w.r.t. $p(x)$, and then compose it with $F^{-1}(x)$ to get $f(x) = g(F^{-1}(F(x))) = g(x)$. I'm not sure under what conditions $g(F^{-1}(y))$ will also be strictly-increasing and concave though.2012-04-02
  • 0
    Is the result true if $\lim_{y \rightarrow \infty} f(y) = \infty$?2012-04-02
  • 0
    I'm not clear about your comment. Of course the result is not true for any such $f(\cdot)$. I'm wondering if there exists at least one. For instance, if $p(x) \sim 1/x^2$ and $\Phi(x) \sim x$ (not integrable), then we can pick $f(y) \sim y^{1/2}$ so that $f(\Phi(x)) \sim x^{1/2}$ and $f(\Phi(x))p(x) \sim 1/x^{3/2}$ (integrable)... moreover, if $\Phi(\cdot)$ is exponential, then we can take $f(\cdot)$ logarithmic... is all that "general"? There exists a proof of that?2012-04-03
  • 0
    I meant $F(\cdot)$ in place of $\Phi(\cdot)$...2012-04-03
  • 0
    @Adam Sorry, I didn't see the "there exists", so my comment, that I removed, didn't make sense.2012-04-03
  • 0
    @ Davide: no problem. I'm searching for such a result (seems to make sense, no?) in my books but I haven't found so far...2012-04-03
  • 0
    Can someone explain to me the answer given in http://mathoverflow.net/questions/93041/composed-function-made-lebesgue-integrable/93046#93046 ??? Thanks!2012-04-04

1 Answers 1

1

Consider the probability measure $m$ with density $p$ on the positive halfline, and any decreasing positive integrable function $c$ on the positive halfline for the Lebesgue measure, for example $c(u)=\mathrm e^{-u}$ for every $u\geqslant0$. You are interested in the finiteness of the integral $$ I(f)=\int f(F(x))\mathrm dm(x). $$ Writing $f(F(x))=\displaystyle\int\limits [u\leqslant f(F(x))]\mathrm du$ and using Fubini theorem, one gets $$ I(f)=\int\mathrm du\int[u\leqslant f(F(x))]\mathrm dm(x)=\int m(A_u)\mathrm du,\qquad A_u=[f(F)\geqslant u]. $$ Define the function $f$ by the identity $$ c(f(t))=m(F\geqslant t), $$ that is, in the example above, $$ f(t)=-\log m(F\geqslant t). $$ The functions $c$ and $t\mapsto m(F\geqslant t)$ are nonincreasing hence $f$ is nondecreasing. Furthermore, $f(0)=0$ and $m(F\geqslant t)\to0$ when $t\to+\infty$ hence $f(t)\to+\infty$ when $t\to+\infty$. Finally, for every positive $u$, $m(A_u)=m(F\geqslant f^{-1}(u))=c(u)$, hence $I(f)=\displaystyle\int c(u)\mathrm du$ is finite.

  • 0
    is it possible to recover a strictly-increasing $f$?2012-04-23
  • 0
    Adam: try! $ $ $ $2012-04-23
  • 0
    Are you saying that your answer already gives $f$ strictly increasing?2012-04-23
  • 0
    Also I'm not clear on the following fact. To apply the Fubini's theorem, the integral must be finite. However we want to prove this, not to assume it. Am I wrong?2012-04-23
  • 0
    Fubini applies to integrals of **nonnegative** functions, whether the integrals are finite or not.2012-04-23
  • 0
    Thanks. I was reading http://en.m.wikipedia.org/wiki/Fubini's_theorem. Can you give a link of the right one (without finite integral)?2012-04-23
  • 0
    Adam: I suggest you read [the very page](http://en.wikipedia.org/wiki/Fubini%27s_theorem#Tonelli.27s_theorem) you linked to.2012-04-24
  • 0
    Both in the Main Statement and in the Alternative one I see that integrability is required... are you sure about your claim?2012-04-24
  • 0
    Quote: *Tonelli's theorem states that on the product of two σ-finite measure spaces, a product measure integral can be evaluated by way of an iterated integral for nonnegative measurable functions, REGARDLESS OF WHETHER THEY HAVE FINITE INTEGRAL.*2012-04-24
  • 0
    Thanks a lot. Addressing the Tonelli's Theorem calling it "Fubini's Theorem" confused me...2012-04-24
  • 0
    I'm still not sure if we can get a strictly-increasing $f$. Your $f$ is in fact only non-decreasing.2012-12-02